Cálculo del conmutador del operador Pauli-Lubanski y generadores del grupo Lorentz

El operador de Pauli-Lubanski se define como

W α = 1 2 ε α β m v PAGS β METRO m v , ( ε 0123 = + 1 , ε 0123 = 1 )
dónde METRO m v es el grupo de generadores de Lorentz.

La relación de conmutación entre generadores del grupo de Poincaré se conoce como

i [ METRO m v , METRO ρ σ ] = η v ρ METRO m σ η m ρ METRO v σ η m σ METRO ρ v + η v σ METRO ρ m ,
i [ PAGS m , METRO ρ σ ] = η m ρ PAGS σ η m σ PAGS ρ .

Intento derivar el conmutador entre el operador de Pauli-Lubanski y un generador del grupo de Lorentz, que también se da en nuestra lección.

(*) i [ W α , METRO ρ σ ] = η α ρ W σ η α σ W ρ .

pero solo obtengo

i [ W α , METRO ρ σ ] = i [ 1 2 ε α β m v PAGS β METRO m v , METRO ρ σ ] = 1 2 ε α β m v PAGS β ( d v ρ METRO m σ d m ρ METRO v σ d m σ METRO ρ v + d v σ METRO ρ m ) + 1 2 ε α β m v ( d β ρ PAGS σ d β σ PAGS ρ ) METRO m v .

Obviamente puedo contratar los deltas, pero eso no me acerca al resultado más simple de (*). ¿Alguien puede señalar qué hacer a continuación?

La estructura de su índice no se ve bien. Por ejemplo, en el último término el m y v los índices son gratuitos pero deben contratarse. También contrata índices donde ambos índices suben. Necesitas usar la relación [ A B , C ] = A [ B , C ] + [ B , C ] A y las propiedades de (anti-)simetría de los índices.
@Heidar: arreglado. Lo hice correctamente en papel borrador, pero me equivoqué al escribir en una computadora.
@Heidar: Traté de usar la antisimetría al principio para simplificar los primeros cuatro términos. Esperaba que se cancelaran entre sí, pero se redujeron a dos términos diferentes. Todavía no está cerca del resultado final.

Respuestas (5)

Tal vez, la respuesta no sea útil para el autor de la pregunta, pero es importante para las otras personas que quieren derivar esta expresión. [:)].

Primer método .

Hice una pista: es fácil mostrar que

W ^ m PAGS ^ m = 1 2 ε m α β γ j ^ α β PAGS ^ γ PAGS ^ m = 0
como la convolución de simétrica PAGS ^ γ PAGS ^ m y antisimétrico ε m α β γ .

Entonces el conmutador

[ j ^ k λ , W ^ m PAGS ^ m ] = 0. ( .1 )
Pero por otro lado
[ j ^ k λ , W ^ m PAGS ^ m ] = W ^ m [ j ^ k λ , PAGS ^ m ] + [ j ^ k λ , W ^ m ] PAGS ^ m = W ^ m i ( gramo m k PAGS ^ λ gramo m λ PAGS ^ k ) + [ j ^ k λ , W ^ m ] PAGS ^ m .
Entonces, usando ( .1 ) de la fórmula anterior se sigue la siguiente:
[ j ^ k λ , W ^ m ] PAGS ^ m = i W ^ m ( gramo m k PAGS ^ λ gramo m λ PAGS ^ k ) = i ( W ^ k PAGS ^ λ W ^ λ PAGS ^ k ) = i ( W ^ k d λ m W ^ λ d k m ) PAGS ^ m ,
dónde d 0 0 = 1 , d i i = 1 ,

y finalmente,

[ j ^ k λ , W ^ m ] = i ( W ^ k d λ v W ^ λ d k m ) [ j ^ k λ , W ^ m ] = i ( W ^ k gramo m λ W ^ λ gramo m k ) .

Segundo método .

Es similar al método QMechanic. Tengamos la matriz de transformación general de Poincare tu ( Λ ^ , a m ) , dónde a es traslación de 4 vectores y Λ ^ es una matriz de 3 rotaciones y transformaciones de Lorentz. Debido a esta transformación,

j ^ m v tu ( Λ ^ , a m ) j m v tu ( Λ ^ , a m ) + = Λ ρ m Λ σ v j ^ ρ σ + a m Λ ρ v PAGS ^ ρ a v Λ ρ m PAGS ^ ρ ,
PAGS ^ m Λ v m PAGS ^ v .
Mediante el uso de estas reglas de transformación se puede obtener
tu ( Λ ^ , a ) W m tu ( Λ ^ , a ) + = 1 2 ε m v ρ σ ( λ α v Λ β ρ j ^ α β + a v Λ α ρ PAGS ^ α a ρ Λ ρ v PAGS ^ ρ ) Λ d σ PAGS ^ d =
= 1 2 Λ m α ε α v ρ σ j ^ v ρ PAGS ^ σ = Λ m α W ^ α .
Asi que
i 2 [ j ^ m v , W ^ σ ] = ω m v gramo σ m W ^ v = 1 2 ( gramo σ m W ^ v gramo σ v W ^ m ) ω m v ,
que conduce a la expresión dirigida.

En general, esto significa que todos los operadores de 4 A ^ γ viaja con j ^ α β como i ( gramo γ α A ^ β gramo γ β A ^ α ) , porque j ^ α β representa a los generadores del grupo Lorentz. Además, esto significa que el conmutador de j ^ α β con cada 4 escalares dará cero.

Si entiendo correctamente, en su primer método, se basa en la afirmación de que A ^ k λ m PAGS ^ m = 0 implica A ^ k λ m = 0 (donde en su caso A ^ k λ m = [ j ^ k λ , W ^ m ] i ( W ^ k d λ m W ^ λ d k m ) ). No creo que esto sea correcto.

He aquí una posible estrategia:

  1. Considere una matriz antisimétrica real arbitraria (infinitesimal)

    (1) ω m v   =   ω v m .

  2. Definir

    (2) ω m λ   :=   ω m v η v λ .

  3. Definir

    (3) METRO   :=   i 2 METRO m v ω v m .

  4. Entonces las ecuaciones de OP se pueden reformular como:

    (4) W α   :=   1 2 ε α β m v PAGS β METRO m v ,
    (5) [ PAGS α , METRO ]   =   PAGS β ω β α ,
    (6) [ METRO m v , METRO ]   =   METRO m λ ω λ v ( m v ) ,
    (7) [ W α , METRO ]   =   ω α β W β . ( )

  5. En particular, tenga en cuenta que la pregunta de OP (v2) se puede reformular como Cómo probar la ec. (7)?

  6. Definir la matriz exponencial

    (8) A α β   :=   ( mi ω ) α β   :=   d β α + ω α β + 1 2 ω α γ ω γ β + O ( ω 3 ) .

  7. Expande la siguiente identidad determinante

    (9) A α α A β β A m m A v v ε α β m v   =   det ( A ) ε α β m v
    a primer orden en ω para deducir la siguiente identidad de traza
    (10) ω α α ε α β m v + ω β β ε α β m v + ω m m ε α β m v + ω v v ε α β m v   =   t r ( ω ) ε α β m v ,
    con la ayuda de la identidad traza determinante
    (11) det ( mi ω )   =   mi t r ( ω ) .
    ecuación (10) puede verse como una versión infinitesimal de la ecuación (9), pero debido a la linealidad de la ecuación. (10) es realmente válido para cualquier 1 finito 4 × 4 matriz.

  8. Usa la antisimetría (1) y la definición (2) para deducir que la traza se desvanece

    (12) t r ( ω )   :=   ω m m   =   0.

  9. Finalmente, use las ecs. (4), (5), (6), (10) y (12) para demostrar la ecuación buscada. (7):

    2 [ W α , METRO ]   =   ε α β m v ( [ PAGS β , METRO ] METRO m v + PAGS β [ METRO m v , METRO ] )
      =   ε α β m v ( PAGS λ ω λ β METRO m v PAGS β METRO v λ ω λ m + PAGS β METRO m λ ω λ v )
      =   ( ω β β ε α β m v + ω m m ε α β m v + ω v v ε α β m v ) PAGS β METRO m v
    (13)   =   ω α α ε α β m v PAGS β METRO m v   =   2 ω α β W β .


1 Alternativamente, se puede establecer la ec. (10) por razonamiento puramente combinatorio, sin la ayuda de la ec. (9).

(8)-(10) parece un poco antinatural. La única propiedad de ω m v es antisimetría, por lo que (10) debería poder derivarse solo de eso.
Obtuve [ ε α β m v PAGS β METRO m v , METRO ] = ε α β m v PAGS λ METRO m v ω λ β + ε α β m v PAGS β METRO m λ ω λ v ε α β m v PAGS β METRO v λ ω λ m sustituyendo en su sugerencia. Perdón por mi estupidez, porque no sé cómo proceder. La ecuación (10) no parece relevante.
Actualicé la respuesta. (Tenga en cuenta que los números de ecuación pueden haber cambiado).

Los primeros cuatro términos de su cálculo desaparecen. Se pueden llevar a la forma (usando la antisimetría del épsilon y de M)

ϵ ρ α β v METRO v σ ϵ σ α β v METRO v ρ Si introduce números para los índices, verá rápidamente que esto se cancela. Los otros dos términos deberían dar el resultado deseado.

El truco para usar para los otros dos términos es (consulte wikipedia para obtener más información)

ϵ a 1 , a 2 , . . . , a norte = ϵ b 1 , b 2 , . . . , b norte d a 1 b 1 . . . d a norte b 1

Los índices en ϵ ρ α β v METRO v σ ϵ σ α β v METRO v ρ se coloca mal.
Ups, de hecho. En mi boceto en una hoja de papel puse todos los índices libres abajo. Ma mal :) Cálculo pasa sin embargo.
No quiero meterme en números para comprobar que bla, bla, se desvanece. Si pego números para verificar esto, simplemente puedo pegar números para verificar (*).

Puede reescribir el vector de Pauli-Lubanski como W m = ( W 0 , W ) y prueba primero las siguientes identidades

PAGS W = 0 , W 0 = PAGS j , W = PAGS 0 j PAGS × k
dónde PAGS , j , y k se definen como (no estoy seguro de si estamos usando las mismas convenciones)
PAGS = { PAGS 1 , PAGS 2 , PAGS 3 } , j = { METRO 32 , METRO 13 , METRO 21 } , k = { METRO 01 , METRO 02 , METRO 03 }
Luego, las pruebas de [ W 0 , METRO ρ σ ] y [ W i , METRO ρ σ ] será mucho más fácil.

Encontré una forma más directa de resolver ......

R . H . S . = gramo m ρ W σ gramo m σ j ρ = 1 2 ϵ ρ σ α β ϵ θ τ α β gramo m θ W τ = 1 4 ϵ ρ σ α β ϵ θ τ α β gramo m θ ϵ τ v yo λ τ PAGS v j yo λ = 1 2 ϵ ρ σ α β ( PAGS m j α β + PAGS α j β β m PAGS β j α α m )
L . H . S = 1 2 ϵ m k yo v m [ PAGS k ( gramo v ρ j yo σ gramo yo ρ j v σ gramo σ yo j ρ v + gramo σ v j ρ yo ) + ( gramo k ρ PAGS σ gramo k σ PAGS ρ ) j yo v ] = ϵ m ρ k yo PAGS k j yo yo σ ϵ m σ k yo PAGS k j yo yo ρ + 1 2 ( ϵ m ρ yo v PAGS σ j yo v ϵ m σ yo v PAGS ρ j yo v ) .
Eso es probar:
1 2 ϵ α β { ρ σ PAGS m } j α β + ϵ α β { ρ σ PAGS α j β β m } = 0.
dónde F { a b C } = F a b C + F b C a + F C a b . obviamente era equivalente a:
ϵ θ k yo τ ϵ θ ρ σ m ϵ α β α β k yo ( PAGS τ j α β + PAGS α j β τ + PAGS β j τ α ) = 0.
del mismo modo:
ϵ θ k yo τ ϵ θ ρ σ m ϵ α β k yo α β ϵ m v λ ω ϵ m τ α β PAGS v j λ ω = 0.
pecado ϵ θ k yo τ ϵ α β k yo α β ϵ m τ α β = 0 , la proposición original ha sido probada.